LSAT and Law School Admissions Forum

Get expert LSAT preparation and law school admissions advice from PowerScore Test Preparation.

 Administrator
PowerScore Staff
  • PowerScore Staff
  • Posts: 8916
  • Joined: Feb 02, 2011
|
#24965
Complete Question Explanation

Weaken—CE. The correct answer choice is (B)

This stimulus presents an ecological theory regarding an ecosystem’s characteristics. The “bottom up” theory suggests that the development of an ecosystem is primarily based on the availability of edible plants, as that determines the number of herbivores the environment can support which in turn determines the number of predators it can support. Thus, the theory is named “bottom up” as it is based on the lowest part of the food chain (the bottom) and builds up to the predators (the top). The author then states that according to the theory, a reduction in the number of predators will have little impact on the rest of the ecosystem. This is our causal statement.
  • Cause ..... ..... ..... ..... ..... ..... ..... Effect

    reduction in # of predators ..... :arrow: ..... little impact on rest of ecosystem
If you look closely at this stimulus, you will notice that there is no overt conclusion stated. Moving on to the question stem, we can recognize that it is a Weaken question as it asks for the answer choice that provides the most evidence against the argument in the stimulus. However the question is a bit more specific; it asks us to weaken the theory that the development of an ecosystem is dependent on plants. Even though the stimulus itself doesn’t give us a conclusion to weaken, the question stem directs our focus.

Answer choice (A): This answer choice actually supports the theory. Since the theory holds that the number of herbivores an environment can support is dependent on the available supply of plants, the monkeys’ demise in an environment lacking adequate food provides an example of how the theory works in practice.

Answer choice (B): This is the correct answer choice. As explained above, the causal reasoning in the stimulus shows that when there is a decrease in the number of predators, there is little impact on the rest of the ecosystem. In this answer choice, there is a decrease in the number on predators, but there is a major impact on the rest of the environment. Therefore, since the cause occurred, but the effect did not, this is the correct answer choice.

Answer choice (C): Much like answer choice (A), this answer choice supports the stimulus. Here, the reduction in plants was followed by a reduction in herbivores (the ants), which in turn was followed by a reduction in carnivores (anteaters). This situation is what we would expect from the stimulus.

Answer choice (D): This answer choice has no impact on the theory. Since the old ferns were replaced with the new ferns, there was little to no net reduction in the number of ferns available for consumption. The theory would expect no real change in the ecosystem.

Answer choice (E): This answer choice does not weaken the theory as the ecosystem is stable with the increase in plant life.
 cpando1995@gmail.com
  • Posts: 18
  • Joined: Jul 07, 2016
|
#30322
Hi Powerscore team,

I was very confused by the stimulus because I thought the statement " the availability of edible plants is what primarily determines an ecosystem's characteristics" was the causal statement the question asked to attack. Could anyone explain how it was the last sentence, instead of the one I mentioned above, the one that I should have focused on? Now that I focused on the last statement, the correct answer stood out to me, but I didn't see how that sentence was the key to solving this question.


Thank you,
Cindel P.
 Claire Horan
PowerScore Staff
  • PowerScore Staff
  • Posts: 408
  • Joined: Apr 18, 2016
|
#30368
You are not wrong in your identification of the causal statement that the question is designed to attack. The sentence about predators having little impact on the ecosystem is derived from the statement that the ecosystem is "primarily determine[d]" by the availability of plants. In other words, the theory is that plants control the ecosystem's characteristics rather than the predators having a top-down effect. (B) is correct in that it weakens the "bottom-up" theory by providing an example of an ecosystem in which plant availability was not the primary determinant of ecosystem characteristics.
 cpando1995@gmail.com
  • Posts: 18
  • Joined: Jul 07, 2016
|
#30370
Thank you so much for making me connect the two statements, Claire!
 andriana.caban
  • Posts: 142
  • Joined: Jun 23, 2017
|
#38198
How did you know this was causal reasoning? Is it because the stimulus said that one thing was determined by the other?
 whardy21
  • Posts: 48
  • Joined: Sep 30, 2018
|
#65406
I had trouble locating the conclusion. I thought when it says "the availability of edible plaints is what primarily determines an ecosystem' characteristics" was the conclusion. Can someone confirm this? I want to know do Cause & Effect questions all of causal conclusions.

I was between B and D and chose D. I wanted to weaken the conclusion that I stated above. I believe my choice was wrong because even though the herbivore population wasn't effected from eating the native ferns, however they thrived on the new species of ferns introduced, which speaks to how the ecoystems's characteristic can determine herbivores. Please check my logic. Thanks.

Answer choice B in my opinion just looks like the relationship is reverse between herbivores species increasing and plants reducing compared to the stimulus.
 Brook Miscoski
PowerScore Staff
  • PowerScore Staff
  • Posts: 418
  • Joined: Sep 13, 2018
|
#65435
Adrian,

In part, yes, the "determination" indicates that the stimulus uses causal reasoning, but also the word "impacted." You can see from the last sentence that the stimulus is arguing that edible plants cause population, predators do not cause population.

Whardy, the stimulus explicitly provides one prediction of the "bottom up theory," which is whether a reduction in the number of predators will have an effect. Thus, you are right about the central conclusion of the theory, but you must remain open to a choice that contests the theory's prediction.

(B) directly contradicts the theory's prediction, which weakens the theory.

(D) is not a strong challenge the stimulus, which is about the plant-herbivore-carnivore structure of the ecosystem, not the specific plants involved. (D) tends to strengthen the structural assertion of the stimulus (number of plants causes number of herbivores causes number of carnivores). You are correct that the ecosystem would have different characteristics, but those weren't the numerical characteristics the stimulus was discussing.

Get the most out of your LSAT Prep Plus subscription.

Analyze and track your performance with our Testing and Analytics Package.